Find CLAT UG 2020 Solved Paper on Legal Bites for the upcoming examination. Attempting these Previous Year Papers will allow you to get a grip on the foundation and nature of the exam. Once you get familiar with the nature and pattern of the exam, you can develop a strategy accordingly. Securing a seat in your favourite law… Read More »

Find CLAT UG 2020 Solved Paper on Legal Bites for the upcoming examination. Attempting these Previous Year Papers will allow you to get a grip on the foundation and nature of the exam. Once you get familiar with the nature and pattern of the exam, you can develop a strategy accordingly.

Securing a seat in your favourite law school is not that easy. Practice CLAT UG Previous Years Papers and get an idea about where you stand today. The score reflects the preparation and gives a chance for you to improve it further. It helps in the evaluation of your knowledge and understanding of concepts and subjects you have studied. Candidates can get a rich understanding of the level of the examination by attempting these papers.

Students preparing for the upcoming CLAT UG Exam can find the Solved Previous Year Papers below.

Common Law Admission Test | CLAT UG 2020 | Click Here to Download PDF

COMMON LAW ADMISSION TEST (CLAT UG) 2020

Duration of Test: 2 Hours (120 minutes)

Maximum Marks: 150

Section I – English

Climate change is considered to be one of the most serious threats to sustainable development, with adverse impact on the environment, human health, food security, economic activity, natural resources and physical infrastructure. According to the Intergovernmental Panel in on Climate Change (IPCC), the effects of climate change have already been observed, and scientific findings indicate that precautionary and prompt action is necessary. Vulnerability to climate change is not just a function of geography or dependence on natural resources; it also has social, economic and political dimensions which influence how climate change affects different groups. Poor people rarely have insurance to cover loss of property due to natural calamines, i.e., drought, floods, super cyclones etc. The poor communities are already struggling to cope with the existing challenges of poverty and climate variability and climate change could push many beyond their ability to cope or even survive. It is vital that these communities are helped to adapt to the changing dynamics of nature. Adaptation is a process through which societies make themselves better able to cope with an uncertain future. Adapting to climate change entails taking the right measures to reduce the negative effect of climate change (or exploit the positive ones) by making the appropriate adjustments and changes. These range from technological options such as increased sea defences or flood proof houses on stilts to behavioural change at the individual level, such as reducing water use in times of drought. Other strategies include early warning systems for extreme events, better water management, improved risk management, various insurance options and biodiversity conservation. Because of the speed at which climate change is happening due to global temperature rise, it is urgent that the vulnerability of developing countries to climate change is reduced and their capacity to adapt is increased and national adaptation plans are implemented. Communities must build their resilience, including adopting appropriate technologies while making the most of traditional knowledge, and diversifying their livelihoods to cope with current and future climate stress. Local coping strategies and knowledge need to be used in synergy with government and local interventions. The need of adaptation interventions depends on national circumstances. There is a large body of knowledge and experience within local communities on coping with climatic variability and extreme weather events. Local communities have always aimed to adapt to variations in their climate. Local coping strategies are an important element of planning for adaptation. Traditional knowledge can help to provide efficient, appropriate and time tested ways of advising and enabling adaptation to climate change in communities who are feeling the effects of climate changes due to global warming.

1. To address the challenge of climate change, developing countries urgently require:

  1. Implementation of National Adaptation Plans
  2. Adoption of short term plans
  3. Adoption of technological solutions
  4. Imposition of climate change tax.

Answer: a.

2. Given below are the factors of vulnerability of poor people to climate changes. Select the option that contains the correct answer.

  1. Their dependence on natural resources.
  2. Geographical attributes.
  3. Lack of financial resources.
  4. Lack of traditional knowledge.

Codes:

  1. (II), (III) and (IV)
  2. (I), (II), (III) and (IV)
  3. (III) only
  4. (I), (II) and (III).

Answer: d.

3. Which of the following is against the idea portrayed in the passage?

  1. Co-ordination between regional and national efforts is necessary
  2. The process of adaptation to climate change does not take into account the factor of prevailing national circumstances
  3. Social dimensions of climate change also need to be appreciated
  4. Combining traditional knowledge with appropriate technology is the need of the hour.

Answer: b.

4. The traditional knowledge should be used through:

  1. Improvement in national circumstances
  2. Synergy between Government and local interventions
  3. Imposition of climate change tax
  4. Its dissemination.

Answer: b.

5. What is the meaning of the word ‘Resilience‘ occurring in the passage?

  1. Toughness
  2. Fragility
  3. Flexibility
  4. Vulnerability.

Answer: c.

The uncovering of a private Instagram group styling itself ‘Bois Locker Room’ featuring students from some prominent South Delhi schools discussing their female classmates in disturbingly violent ways including plans of sexual assault is a wakeup call for parents and authorities. The group formed last month or so kicked up a social media storm when screenshots surfaced. Police have questioned a 15 year old boy to identify other members. Similar incidents involving minors discussing rape/ gang rape of classmates have been reported on other digital platforms like WhatsApp too, across cities. The exchanges in the now deleted group require precise responses from police, parents and school authorities around whom the fates of the juveniles involved now revolve. It is important to recognise where a teenager spouting objectification of his female counterparts is coming from. People of all ages, not just children, are retreating deeper into the recesses of their online avatars during this lockdown. But the heavy technological investment in children‘s education, including flooding them with personal smartphones, has not been matched by serious conversations centred on responsible internet usage and equality. Young, impressionable minds absorb the normalisation of rape from the adults around them. When what they see, read and hear is toxic masculinity, that is what they perform. That‘s what peer pressure becomes about. But if this youthful role play of macho dominance receives timely counselling, it can prevent far graver adult offences. Schools and parents have a critical role to play in educating children on gender equality. Digital platforms which claim to have zero tolerance towards content that violates community standards must also explain why such abuses go undetected, despite boasts about Artificial Intelligence-driven technologies to stop them. They should play a more proactive role in stopping the sexual harassment of real people in the guise of virtual sport.

[Editorial Published in Times of India, dated 6 May, 2020]

6. What is the central idea being conveyed by the Author in the passage above?

  1. Modern Social Networking platforms must take the lead in regulating young people from using these platforms for sexual abuse
  2. It is significant to educate children and youngsters about gender equality
  3. Digital platforms need to have a zero tolerance policy towards content leading to sexual abuse
  4. None of the above.

Answer: a.

7. As per author, why is this incident a wake-up call for parents?

  1. Because even the parents are indulged heavily in social networking platforms
  2. Because parents have failed to stop their children from committing such blunders
  3. Because it is the primary responsibility of parents to control their children
  4. Because the parents have failed in educating their children regarding fair internet usage and have simply invested in the technology driven education of their children.

Answer: d.

8. According to the author, in order to understand the root cause of such insensitive incidents, it is important to:

  1. Understand and examine the kind of socialisation process these children are exposed to
  2. Adopt Artificial Intelligence driven technologies
  3. Strengthen the environment of discipline in schools and colleges
  4. All of the above.

Answer: d.

9. On the basis of your reading of the passage, which of the following statements can be inferred?

  1. Sexual offences can be reduced by timely counselling from parental and quasi-parental authorities, i.e., parents and teachers respectively
  2. If digital platforms implement a Zero Tolerance Policy towards such a menace, sexual offenses can be completely stopped
  3. Investment in education of children is enough to curb the menace
  4. All of the above.

Answer: a.

10. Which of the following words are synonymous with the word ‘Spouting‘?

  1. Pouring
  2. Streaming
  3. Erupting
  4. All of the Above.

Answer: d.

The snow was falling, and the Cat’s fur was stiffly pointed with it, but he was imperturbable. He sat crouched, ready for the death-spring, as he had sat for hours. It was night but that made no difference, all times were as one to the Cat when he was in wait for prey. Then, too, he was under no constraint of human will, for he was living alone that winter. Nowhere in the world was any voice calling him; on no hearth was there a waiting dish. He was quite free except for his own desires, which tyrannized over him when unsatisfied as now. The Cat was very hungry. almost famished, in fact. For days the weather had been very bitter…and the Cat’s long hunt had availed him nothing. But he waited with the inconceivable patience and persistency of his race; besides, he was certain. The Cat was a creature of absolute convictions, and his faith in his deductions never wavered. The rabbit had gone in there between those low-hung pine boughs. The Cat had seen her enter…so he sat down and waited, and he waited still in the white night, listening angrily to the north wind starting in the upper heights of the mountains with distant screams, then swelling into an awful crescendo of rage, and swooping down with furious white wings of snow like a flock of fierce eagles into the valleys and ravines. The Cat was on the side of a mountain, on a wooded terrace. Above him, a few feet away towered the rock ascent as steep as the wall of a cathedral. He had often looked with wonder at the rock, and miauled bitterly and resentfully as man does in the face of a forbidding Providence. At his left was the sheer precipice. Behind him…was the frozen perpendicular wall of a mountain stream. Before him was the way to his home. When the rabbit came out she was trapped; her little cloven feet could not scale such unbroken steeps. So the Cat waited. The tangle of trees and bushes clinging to the mountain-side with a stern clutch of roots, the prostrate trunks and branches, the vines embracing everything with strong knots and coils of growth, had a curious effect, as of things which had whirled for ages in a current of raging water, only it was not water, but wind, which had disposed everything in circling lines of yielding to its fiercest points of onset. It was as if ice needles pricked his skin through his beautiful thick fur, but he never faltered and never once cried. He had nothing to gain from crying, and everything to lose; the rabbit would hear him cry and know he was waiting.

[Excerpts from a Short story, “The Cat” by Mary E. Wilkins Freeman]

11. Which of the following suggests a synonymous meaning to the words ‘Providence‘ and ‘Crescendo‘ respectively?

  1. Nemesis, Apex
  2. Zenith, Nadir
  3. Laxity, Prudence
  4. Short-sightedness, Upsurge.

Answer: a.

12. The passage has been adorned with numerous figure of speeches. Which of the following combinations is correct?

  1. Irony and Sarcasm
  2. Alliteration and Pun
  3. Simile and Personification
  4. Metaphor and Onomatopoeia.

Answer: c.

13. The passage best demonstrates which one of the following motifs of Cat‘s Life?

  1. To satisfy the pangs of hunger
  2. To survive the harsh winters
  3. A never ending wait
  4. To hunt for Rabbit.

Answer: c.

14. The Author‘s description of “…he was under no constraint of human will, for he was living alone…” implies:

  1. Cat‘s absolute freedom from everything
  2. Cat‘s no association with human beings
  3. Cat‘s loneliness
  4. Cat‘s tyrannical demeanour.

Answer: c.

15. The lines, “…but he never faltered and never once cried. He had nothing to gain from crying, and everything to lose…”, suggest that the Cat is:

  1. Reflective
  2. Apologetic
  3. Resilient
  4. Frustrated.

Answer: c.

I am losing my interest in human beings; in the significance of their lives and their actions. Someone has said it is better to study one man than ten books. I want neither books nor men; they make me suffer. Can one of them talk to me like the night – the Summer night? Like the stars or the caressing wind?

The night came slowly, softly, as I lay out there under the maple tree. It came creeping, creeping stealthily out of the valley, thinking I did not notice. And the outlines of trees and foliage nearby blended in one black mass and the night came stealing out from them, too, and from the east and west, until the only light was in the sky, filtering through the maple leaves and a star looking down through every cranny.

The night is solemn and it means mystery.

Human shapes flitted by like intangible things. Some stole up like little mice to peep at me. I did not mind. My whole being was abandoned to the soothing and penetrating charm of the night. The katydids began their slumber song: they are at it yet. How wise they are. They do not chatter like people. They tell me only: “sleep, sleep, sleep.” The wind rippled the maple leaves like little warm love thrills. Why do fools cumber the Earth! It was a man‘s voice that broke the necromancer‘s spell. A man came today with his “Bible Class.” He is detestable with his red cheeks and bold eyes and coarse manner and speech. What does he know of Christ? Shall I ask a young fool who was born yesterday and will die tomorrow to tell me things of Christ? I would rather ask the stars: they have seen him.

[Short Story by Kate Chopin]

16. Why has the author lost interest in human beings?

  1. Because they make the author suffer
  2. Because the human beings do not bring with them the warmth and the comfort which comes naturally with the night, starts and the wind
  3. Because human beings are not mysterious
  4. All of the above.

Answer: d.

17. The author has compared the night with:

  1. The Katydids
  2. The Necromancer‘s spell
  3. Stars
  4. All of the above.

Answer: b.

18. Why has the author called the ‘katydids‘ wise?

  1. Because they sing a slumber song
  2. Because they tell the author to only ‘sleep‘
  3. Because they do not indulge in unnecessary and unimportant talks
  4. Because they are not fools.

Answer: c.

19. Which of the following can be inferred from the passage?

  1. Being close to nature can bring one closer to God
  2. Nature is just a mystery
  3. Books and man are both detestable
  4. None of these.

Answer: a.

20. Which of the following words from the passage mean ’rough‘?

  1. Solemn
  2. Caressing
  3. Coarse
  4. Slumber.

Answer: c.

It was the strangest murder trial I ever attended, where the old woman was found battered to death. He was a heavy stout man with bulging bloodshot eyes. All his muscles seemed to be in his thighs. The clock had just struck two in the morning. Mrs Salmon in 15 Northwood Street had been unable to sleep: she heard a door click shut and thought it was her own gate. So she went to the window and saw Adams (that was his name) on the steps of Mrs Parker’s house. He had just come out and he was wearing gloves. He had a hammer in his hand and she saw him drop it into the laurel bushes by the front gate. But before he moved away, he had looked up at her window. The fatal instinct that tells a man when he is watched exposed him in the light of a streetlamp to her gaze-his eyes suffused with horrifying and brutal fear, like an animal’s when you raise a whip.

Mrs Salmon was called in the Court.

‘And do you see the man here in Court?’

[She looked straight at the big man in the dock, who stared hard at her with his Pekingese eyes without emotion.] 'Yes,’ she said, 'there he is.’

‘You are quite certain?’

She said simply, ‘I couldn’t be mistaken, sir.’

‘Thank you, Mrs Salmon.’

[Counsel for the defence rose to cross-examine.]

‘Now, Mrs Salmon, you must remember that a man’s life may depend on your evidence.’

‘I do remember it, sir.’

‘Is your eyesight good?’

‘I have never had to wear spectacles, sir.’

‘You are a woman of fifty-five?’

‘Fifty-six, sir.’

‘And the man you saw was on the other side of the road?’

‘Yes, sir.’

‘And it was two o’clock in the morning. You must have remarkable eyes, Mrs Salmon?’ ‘No, sir. There was moonlight, and when the man looked up, he had the lamplight on his face.’

‘And you have no doubt whatever that the man you saw is the prisoner?’

‘None whatever, sir. It isn’t a face one forgets.’

Then he said, ‘Do you mind, Mrs Salmon, examining again the people in Court?

No, not the prisoner. Stand up, please, Mr Adams!

And there at the back of the Court with thick stout body and muscular legs and a pair of bulging eyes, was the exact image of the man in the dock.

‘Now think very carefully, Mrs Salmon. Can you still swear that the man you saw drop the hammer in Mrs Parker’s garden was the prisoner and not this man, who is his twin brother?’ Of course she couldn’t.

[Excerpts from “The Case for the Defence” by Graham Greene]

21. Why do you think, has the Author called the trial, ‘strangest‘ murder trial he ever attended?

  1. Because the accused had a very strange personality
  2. Because, despite having a potential witness and evidence against the accused, his wrong could not be proved
  3. Because, Mrs. Salmon‘s evidence could not be proved before the Court
  4. None of these.

Answer: b.

22. “…his eyes suffused with horrifying and brutal fear, like an animal’s when you raise a whip.” can be called as:

  1. Personification
  2. Simile
  3. Onomatopoeia
  4. None of the above.

Answer: b.

23. The expression “Pekingese eyes” used in the passage refers to which of the following?

  1. Bulging eyes
  2. Red eyes
  3. Small eyes
  4. None of them.

Answer: a.

24. Why was Mrs. Salmon convinced that the man she saw had committed the crime?

  1. Because she saw the man on the steps of Mrs. Parker‘s House and he was wearing gloves
  2. Because he had a hammer in his hand and she saw him drop it into the laurel bushes by the front gate
  3. Because when he looked up at her window, his eyes were suffused with horrifying and brutal fear
  4. Because she had seen him clearly in the light of street lamp.

Answer: d.

25. Who was murdered in the scene described in the passage?

  1. Wheeler
  2. Parker
  3. Salmon
  4. It cannot be inferred.

Answer: d.

The world has very few devices left to fight COVID-19 with, but technology remains one of them. Whether it is employing the state-of-the-art technology in the discovery of cures or vaccines, or traditional technology services to enhance health care and consultations, or even tools that keep people at home occupied/productive, it is clear that technology will serve humanity at one of its darkest moments. The pandemic has contributed, in no small measure, to the understanding of the myriad ways in which available technologies have not been put to better use, and presented people with multiple opportunities to harness these devices, techniques and methods to get on with life in the time of lockdown. Among the primary uses is telemedicine, rendered inexorable now, by the temporary paralysis brought on by a freeze on movement. The Centre‘s recent guidelines allowing for widespread use of telemedicine services came as a shot in the arm for telehealth crusaders in the country, among them the Telemedicine Society of India that has long been battling to use the technology in its complete arc to reach remote areas in India. This move finds consonance with the rest of the world where several nations, also deeply impacted by the pandemic, have deployed telemedicine to reach people who have been unable to come to hospital, to reduce footfalls in hospitals, and to even provide medical and mental health counselling to countless people. It was way back in 2000 that telemedicine was first employed in India, but the progress has been excruciatingly slow, until the pandemic. However, it does seem as if the medical community was only held back by the lack of legislation to enable tele consultations. Forno sooner was the policy announced, than hospitals and clinicians hurried to jump onto the bandwagon, advertising contact information for patients. The advantages are peculiar in the current context, when putting distance between people is paramount, as tele consultations are not barred even when health care professionals and patients may have to be quarantined. The advancement of telecommunication capabilities over the years has made the transmission of images and sound files (heart and lung sounds, coughs) faster and simpler. Pilot telemedicine experiments in ophthalmology and psychiatry have proven to be of immense benefit to the communities. Telemedicine‘s time is here, finally. While unleashing the full potential of telemedicine to help people, experts and government agencies must be mindful of the possible inadequacies of the medium, and securing sensitive medical information; such cognisance should guide the use of the technology.

[From an Editorial published in “The Hindu” on April 17, 2020]

26. Which one of the following words from the passage means ‘unavoidable’?

  1. Inexorable
  2. Myriad
  3. Excruciatingly
  4. Bandwagon.

Answer: a.

27. Which of the following best summarizes the main idea of the passage?

  1. It is time to unleash the full potential of telemedicine
  2. It is time to unleash the full potential of telemedicine with cognisance of its possible flaws
  3. Technology has not been utilised to its fullest to suit the current needs
  4. People should not be skeptical about the advantages of telemedicine.

Answer: b.

28. Which of the following is a significant factor contributing in slow employment of Telemedicine in India?

  1. Telemedicine was never employed in India
  2. There was an absence of regulations regarding the use of telemedicine consultations and medication systems
  3. Telemedicinal Society of India failed in its efforts to promote the importance of such a system in the country
  4. All of the above.

Answer: b.

29. Telecommunication based medication has the potential for an easy outreach because:

  1. Pilot experiments in telemedicine have been proved to be successful
  2. It helps in easier communication of medical images from one place to another
  3. It helps in securing sensitive medical information
  4. All of the above.

Answer: d.

30. What is the meaning of the expression ‘Shot in the arm’ used in the passage?

  1. Hitting the nail on its head
  2. Bull‘s Eye
  3. Positive impact on something
  4. All of the above.

Answer: c.

Section II – Current Affairs Including General Knowledge

Prime Minister Narendra Modi on Saturday said major global firms are looking at India as a major investment destination, which is reflected by a robust inflow of Foreign Direct Investment (FDI) last financial year, and through ‘Atmanirbhar Bharat Abhiyan‘ (Self-Reliant India initiative) the country is shifting its focus from ’Make in India‘ to ‘Make for world‘. He said Independent India should be “vocal for local” and asked citizens to glorify Indian products to promote ’Atmanirbhar Bharat‘. Unveiling his vision of a Self-Reliant India, the Prime Minister said that the government has unveiled over Rs 110 lakh crore National Infrastructure Pipeline (NIP) to boost the economy and create jobs. “In order to rapidly modernise India, there is a need to give a new direction to overall infrastructure development,” he said, adding that over 7,000 projects under NIP have been already identified. “This will be, in a way, a new revolution in the field of infrastructure. This is the time to end silos in infrastructure. There is a plan to connect the entire country with multimodel connectivity infrastructure,” he said. NIP will play a crucial role in overcoming the adverse impact of Covid-19 on the economy and catapult the economy in a higher growth trajectory, he said. The government on December 31 last year unveiled the NIP with an aim to make India a $5 trillion economy by 2024-25. The focus of the infrastructure pipeline is to accelerate growth and create employment in both urban and rural areas.

Source: Excerpt from Hindustan Times, written by Rajeev Jayaswal. (Dated 15th August, 2020)

31. The Government announced a stimulus package under the Self-Reliant India Scheme for the amount of ………………

  1. 2 trillion
  2. 20 trillion
  3. 5 trillion
  4. 100 trillion.

Answer: b.

32. Which of the following is not a goal of the stimulus package announced under the “Atmanirbhar” India?

  1. It will help in alleviating the distress of migrants when they return to their villages
  2. It will help in achieving the goal of a self-sustainable rural economy
  3. It would boost the One Nation One Market objective and help India to become the food factory of the world
  4. It will help in elevating quality of life in urban areas.

Answer: d.

33. Which of the following events can be seen as the stepping stone for the growth of the Foreign Direct Investment (FDI) in India?

  1. The launch of ‘Make in India‘ initiative in 2014
  2. The Economic liberalisation in the year 1991
  3. The Amendment in the FDI policy to increase the upper cap from 26% to 49%
  4. All of the above.

Answer: b.

34. Which of the following is not related to the “Vocal for Local” initiative of the Indian Government?

  1. Rebooting the MSMEs especially the Khadi and Village industries
  2. Generating new opportunities of employment at the local level
  3. Creating new economic hubs through disinvestment and FDIs
  4. Promoting the indigenous manufacture and support through financial aid.

Answer: c.

35. Under the National Infrastructure Pipeline, the Government plans to invest more than Rs. 102 lakh crore on infrastructure projects by 2024-25, with the Centre, States and the private sector to share the capital expenditure. What is the ratio of such expenditure by each of the stake holders?

  1. 50:25:25
  2. 40:40:20
  3. 39:39:22
  4. 34:33:33.

Answer: d.

The central bank doesn‘t disclose its foreign exchange management strategy, but it was evident in the last few years that the rupee was not allowed to appreciate despite healthy inflows, resulting in a rapid build-up of foreign exchange. From a low of $275 billion in September of 2013, when rupee came under severe pressure due to so-called ‘taper tantrums‘ by the US Federal Reserve, India now has record foreign exchange reserves of [1] billion, as on 21 August — a 95 per cent rise over seven years. Despite the Covid-19 pandemic, the foreign exchange kitty swelled by $62 billion since March. In this seven-year period, rupee ended the year with an appreciation against the dollar only once — in 2017. This year, the rupee is so far down by 2.04 per cent against the dollar. The latest RBI statement suggested that it is not uncomfortable with the appreciation in rupee, confirming the speculation among currency analysts that a departure was made in the exchange management policy.

The Reserve Bank of India (RBI) said that it will conduct liquidity operations worth Rs 20,000 crore in two tranches through sale and purchase of government securities (G-Secs). The two open market operations (OMOs) of Rs 10,000 crore each will be conducted on September 10 and 17, the central bank said in an official release. This is now the second such announcement in as many weeks. Last week, RBI had announced sale and purchase of GSecs worth Rs 20,000 crore, in two tranches, slated to be conducted on August 27 andSeptember 3. In another move, RBI announced the infusion of Rs 1 lakh crore in mid-September through long-term repo operations (LTROs) at floating rates, or the prevailing repo rate. Moreover, the central bank also gave an option to lenders who have earlier availed funds through LTROs, to reverse their transactions before maturity.

Source: Excerpt taken from the Print.in, written by Manojit Saha. (Dated 2nd September, 2020)

36. Which of the following has been replaced by [1] in the above passage?

  1. $ 537
  2. $ 498
  3. $ 502
  4. $ 756.

Answer: a.

37. Which of the following economic policies have been recently used by the RBI to boost the economy during the current Pandemic situation?

  1. FOREX SWAP
  2. LTROs (Long Term Repo Operations)
  3. OMOs (Open Market Operations)
  4. All of the above.

Answer: d.

38. Which of the following statements relate to the Long Term Repo Operations as measure to support the economy in corona pandemic situations?

  1. It is a measure that is expected to bring down short-term rates and also boost investment in corporate bonds
  2. It would encourage banks to undertake maturity transformation smoothly and seamlessly so as to augment credit flows to productive sectors
  3. It will enhance liquidity in the banking system by Rs 1 trillion
  4. All of the above.

Answer: d.

39. Which of the following will be used as a platform for core banking solutions to implement the LTRO?

  1. E-cartel
  2. E-kuber
  3. E-CBS
  4. E-RBICBS.

Answer: b.

40. What is the notified amount of sum offered by the RBI under measures of the Foreign Exchange Swap?

  1. $ 5 billion
  2. $ 16.71 billion
  3. $ 16.21 billion
  4. $ 5.02 billion.

Answer: a.

The first batch of five Rafale jets flew out of France on July 25 and will arrive in India on Wednesday, July 29, when the fighter jets will officially be inducted and join the Indian Air Force fleet in Haryana’s Ambala. The fighter jets, built by French aviation firm [1], took off from the Merignac airbase in southern France’s Bordeaux today and landed safely at Al Dhafra airbase in the UAE after a sortie of more than seven hours. The five aircraft will be the first tranche of the 36 planes bought by India from France in a Rs. 59,000-crore intergovernmental deal in 2016. “Delivery of ten aircraft has been completed on schedule. Five will stay back in France for training Mission. The delivery of all thirty six aircraft will be completed on schedule by the end of 2021,” the Indian embassy in France said in a statement.

The Rafale fighter jets are capable of carrying a range of highly effective weapons, including the Meteor air-to-air missile and Scalp cruise missile. The Rafale jets will come with various India-specific modifications, including Israeli helmet-mounted displays, radar warning receivers, low-band jammers, 10-hour flight data recording, infra-red search and tracking systems among others. The Air Force has readied the required infrastructure to welcome the jets in its line-up.

Source: Excerpt taken from the NDTV, Reported by Vishnu Som, Edited by Shylaja Varma. (Dated 27th July, 2020)

41. The Rafale deal between India and France is seen to carry great strategic significance. Which of the following is directly impacted by this deal?

  1. The Joint Strategic Vision of India-France Cooperation in the Indian Ocean Region
  2. The one belt one road mission of the China
  3. The Chinese initiative to build a new Silk route
  4. The trade route of the Middle East.

Answer: a.

42. The Indian Air Force is also in the process of procuring a new generation medium-range modular air-to-ground weapon system Hammer to integrate with the Rafale jets. What does the term Hammer stands for?

  1. Highly Advanced Modular Mutation Extended Range
  2. Highly Agile Modular Munition Extended Range
  3. Highly Advanced Modular Mitigation Extended Rafale
  4. Highly Agile Modular Munition Extended Rafale.

Answer: b.

43. Which of the following has been replaced by [1] in the passage?

  1. LAMSCO
  2. LUAD Defence France
  3. DASSAULT
  4. BORESCOPE.

Answer: c.

44. Which of the following indigenous organisations is responsible for the design, assembly and fabrication of combat aircrafts in India?

  1. DRDO
  2. HAL
  3. DRAI
  4. ISRO.

Answer: b.

45. Which of the following organisations/institutions is a major offset partner of India in the Rafale Deal?

  1. Hindustan Aeronautics Limited
  2. Reliance Defence Limited
  3. Reliance Naval and Engineering Limited
  4. Pipavav Defence.

Answer: b.

On May 8, India‘s Defence Minister virtually inaugurated a new 80 km-long road in the Himalayas, connecting to the border with China, at the Lipulekh pass. The Nepali government protested immediately, contending that the road crosses territory that it claims and accusing India of changing the status quo without diplomatic consultations. Among the many escalatory moves since then, Nepal deployed police forces to the region, summoned the Indian ambassador in Kathmandu, and initiated a constitutional amendment to formalise and extend its territorial claims over approximately 400 sq km. India, on the other hand, has conveyed its openness to a dialogue but does not seem to share Nepal‘s sense of urgency: its initial statement agreed to a dialogue, but only after the COVID-19 crisis. India has been in effective possession of this territory for at least sixty years, although Nepal claims it conducted a census there in the early 1950s and refers to the 1815 Sugauli Treaty as legitimising its claims. But India‘s new road, up to the Lipulekh pass, is not an unprecedented change in the status quo. India has controlled this territory and built other infrastructure here before, besides conducting its administration and deploying military forces up to the border pass with China. The region is of strategic importance, and the new road is now one of the quickest links between Delhi and the Tibetan plateau. In a 2015 statement, China also recognised India‘s sovereignty by agreeing to expand trade through the Lipulekh pass.

Source: Excerpt from the brookings.edu, written by Dr. Constantino Xavier. (Dated 11th June, 2020)

46. Which of the following incidents can be seen as the direct cause of the border dispute between India and Nepal?

  1. The Defence Minister inaugurated a motor-able link road that connects India and China
  2. The 2015 agreement between India and China for using the Lipulekh pass for trade
  3. India published a new map which showed the region of Kalapani as part of the Indian Territory
  4. All of the above.

Answer: d.

47. The confusion between Nepal and India consists of a dispute over a trijunture of ……………

  1. Kali – Susta – Ganga
  2. Kalapani – Saraswati – Ganga
  3. Kalapani – Limpiyadhura – Lipulekh
  4. Kalapani – Limpiyadhura – Kali.

Answer: c.

48. The treaty of Sugauli was signed between which of the following pairs?

  1. India and Nepal
  2. United Kingdom, India and Nepal
  3. India, China and Nepal
  4. East India Company and Nepal.

Answer: d.

49. Recently Nepal has issued a map claiming Lipulekh pass as part of its territory included in the district of:

  1. Kailali
  2. Darchula
  3. Dipayal
  4. Doti.

Answer: b.

50. Which of the following rivers is called as Narayani River in Nepal?

  1. Bramhaputra
  2. Gandak
  3. Ganga
  4. Kali.

Answer: b.

One thing struck us as a major difference between the new National Education Policy (NEP) 2020 and its predecessor. The earlier national policies on education (NPE) from 1986 and 1992 presented themselves as attempts to consolidate and build on earlier efforts, particularly the NPE, 1968. The new NEP 2020 policy, on the other hand, is very keen to establish that it is different from everything in the past, including in its name. Nowhere does this attitude come across as starkly as it does in the section on higher education.

It comes across fairly clearly on how the higher education ecosystem will be by 2040. By this time — if the policy has its way — the Indian higher education ecosystem will be populated with higher education institutions (HEI). These will comprise Universities and Colleges and the public and private sectors, all of which will be ’multi-disciplinary‘, with each populated by more than 3,000 students, with at least one “in or near every district”. Universities will conduct research and post-graduate and under-graduate teaching, some research-intensive and others teaching-intensive. Colleges will largely teach at the under-graduate level, with a number of them having their medium of instruction in either bilingual or local / Indian languages. The colleges can manifest in clusters around universities as constituent colleges or may be standalone autonomous ones. Ideally, all HEIs will eventually become “independent self-governing institutions” with considerable”‘faculty and institutional autonomy”. They will have complied with a series of regulatory exercises that are “light-but-tight” and will be operated by a large number of private accreditors, overseen by a new set of regulatory institutions at the national level.

Source: Excerpt taken from downtoearth.org.in, written by Nitin Mehta & Gagan Mehta.(Dated 14th August, 2020)

51. Which of the following statements are not true in relation to the changes envisioned by the NEP 2020?

  1. The NEP 2020 aims at making India a global knowledge superpower
  2. The renaming of the Ministry of Human Resource Development to the Ministry of Education
  3. The development of National Course Curriculum to provide new curriculum by 2021
  4. The New Education Policy aims to facilitate an inclusive, participatory and holistic approach.

Answer: c.

52. The NEP 2020 aims to provide a holistic change to the current education system in India. Which of the following is not related to the aim of NEP 2020?

  1. Universalization of education from preschool to secondary level with 100% Gross Enrolment Ratio
  2. To bring 20 million out of school children back into the mainstream through an open schooling system
  3. Vocational Education to start from Class 6 with Internships
  4. To achieve 80% Gross Enrolment Ratio at the under graduate level.

Answer: d.

53. Which of the following is proposed as a change in the structure of institutions responsible for affiliation and accreditation under the NEP 2020?

  1. Higher Education Commission of India (HECI) will be set up as a single umbrella body for the entire higher education
  2. University Grant Commission to be renamed as AICTE
  3. National Affiliation and Accreditation Council to be renamed as National Standard Council of India
  4. National Council for Education Research and Teaching to be renamed as BCERT.

Answer: a.

54. Which of the following is not a vertical under the Higher Education Commission of India (HECI)?

  1. National Higher Education Regulatory Council (NHERC)
  2. Higher Education Grants Council (HEGC)
  3. National Educational Council (NEC)
  4. National Accreditation Council (NAC).

Answer: c.

55. What is the proposed percentage of expenditure for education out of GDP under the NEP 2020?

  1. 11 percent
  2. 6 percent
  3. 4.6 percent
  4. 9 percent.

Answer: b.

Days after India-Pakistan tensions spilled over into a meeting of the Shanghai Cooperation Organisation (SCO), External Affairs Minister S. Jaishankar and Pakistan Foreign Minister Shah Mehmood Qureshi are expected to meet via a video conference at the South Asian Association for Regional Cooperation (SAARC) meeting on September 24. “All member countries have confirmed participation in the meeting, to be chaired by Pradeep Kumar Gyawali, [1] of Nepal. The respective Foreign Ministers will take part,” sources familiar with preparations for the meeting told The Hindu, referring to the eight members of SAARC, including [2], Bangladesh, [3], India, [4], Nepal, Pakistan and Sri Lanka. A senior Indian official also confirmed that Mr. Jaishankar will attend despite the incident at the SCO virtual meeting of National Security Advisors on Tuesday. During that meeting, National Security Advisor Ajit Doval stormed out after he saw that the Pakistan Special Advisor on National Security Moeed Yusuf had used a map of Pakistan that claimed Indian Territory.

“This was in blatant disregard to the advisory by the host [5] against it and in violation of the norms of the meeting. After consultation with the host, the Indian side left the meeting in protest at that juncture,” the MEA had said about the incident. When asked, the sources said that no specific guidelines on background or maps have been issued by the SAARC Secretariat in Kathmandu that is also the Chair of the SAARC at present, but they hope it would go “smoothly”. A meeting of SAARC Finance Ministers, where an Additional Secretary represented India instead of Finance Minister Nirmala Sitharaman, and Pakistan was represented by its Special Advisor on Finance, took place on Wednesday without incident.

Source: Excerpt taken from The Hindu, written by Suhasini Haider. (Dated 17th September, 2020)

56. Which of the following has been replaced by [1] in the passage?

  1. Prime Minister
  2. Minister of Finance
  3. Minister of External Affairs
  4. Minister of Home Affairs.

Answer: c.

57. Which of the following Nations is part of the South Asian Association for Regional Cooperation?

  1. Indonesia
  2. Thailand
  3. Myanmar
  4. Maldives.

Answer: d.

58. Which of the following has been replaced by [5] in the passage?

  1. Russia
  2. China
  3. Japan
  4. Pakistan.

Answer: a.

59. India had not agreed to attend a summit hosted in Pakistan, but it remains an important promoter of the SAARC process. In which of the following years the SAARC leaders‘ summit was held lately?

  1. 2018
  2. 2014
  3. 2016
  4. 2019.

Answer: b.

60. The above passage mentions about Pakistan‘s Special Advisor on National Security Moeed Yusuf using a map of Pakistan claiming Indian Territory. Which of the following territory is in context here?

  1. Junagadh
  2. Aksai Chin
  3. Pangong Tso
  4. Limpiyadhura.

Answer: a.

In the middle of a pandemic, the geopolitics of the world‘s most troubled region took a historic turn this week, when the UAE and Israel, under the benevolent gaze of US President Donald Trump, signed an agreement to “normalise” relations. The deal opens up new opportunities for India to play a much larger role in the regional security and stability in the Gulf, where New Delhi enjoys special relations with both Abu Dhabi and Jerusalem. The barebones of the deal envisages establishing regular diplomatic relations between the UAE, the rising influential power in the Gulf, and Israel, the “Incredible Hulk” of the region, but a country officially not on speaking terms with most of its Arab neighbours. In his first tweet, Crown Prince Mohammed bin Zayed said: “During a call with President Trump and Prime Minister Netanyahu, an agreement was reached to stop further Israeli annexation of Palestinian territories. The UAE and Israel also agreed to cooperation and setting a roadmap towards establishing a bilateral relationship.” In return, Israel agreed to “suspend” its annexation plans for West Bank that would have been deeply destabilising. Benjamin Netanyahu gets a diplomatic victory, which may be short-lived, given the nature of Israeli politics. But Israel gets a diplomatic and economic opening with the big power in the Gulf that could open other doors, give its security interests legitimacy and, perhaps, open the door to Middle East peace.

Many of the other Arab powers, such as Oman, Bahrain, Egypt and Jordan, apart from the big global powers, and India, have welcomed the deal. Iran has slammed it, as have Turkey and Syria. Saudi Arabia has been very quiet. Given the close ties between Mohammed bin Zayed and Mohammed bin Salman, it is unthinkable that KSA was not consulted, particularly when the US is the third pole in this agreement. The deal gives UAE pole position as the premier

Gulf Arab power, with diplomatic leverage with Israel and the US. “This deal is about positioning in Washington, DC,” said James Dorsey, Gulf and Middle East expert.

Source: Excerpt from the Economic Times, written by Indrani Bagchi. (Dated 16th August, 2020)

61. The US is seen as a major role player in the peace deal between Israel and UAE. Which of the following is not a reason for the involvement of US in the deal?

  1. The deal grants a diplomatic win to the US President Donald Trump ahead of the Presidential election
  2. The efforts of the USA to bring the war in Afghanistan to an end have not been a success yet
  3. The efforts of USA to bring peace between Israel and the Palestinians have not been successful yet
  4. The deal gives a central strategic rule to USA in the Middle East.

Answer: d.

62. Which of the following is true in relation to the stake of Palestine in the peace deal between UAE and Israel?

  1. Israel has recognised Palestine as an Independent Nation
  2. This can be an opportunity for the Palestine to establish diplomatic relations with Israel
  3. The deal marks both a win and setback to the Israel-Palestine relations
  4. A peace treaty would also be signed between Israel and Palestine.

Answer: c.

63. Which of the following is not a direct impact of the peace deal in the Middle East region?

  1. The deal smoothens the UAE‘s international campaign to be seen as a beacon of tolerance in the Middle East
  2. It puts the UAE as leader in a regional power dynamics among neighbouring Gulf Arab states
  3. The deal gives Netanyahu a domestic boost at a time when Israel‘s coalition government is in crisis due to coalition compulsions
  4. The deal marks a historic day and a significant step towards peace in the Middle East.

Answer: d.

64. The peace deal between Israel and UAE can also be seen as a great opportunity for India in the Middle East region. Which of the following is not related to strategic goals of India in the Middle East?

  1. India should revamp its defence and security relations with the UAE
  2. India should move fast to capture the extended neighbourhood market before it comes under the influence of China
  3. India should ensure space for Iran in any future security deal in the region
  4. None of the above.

Answer: c.

65. Which of the following is correct?

  1. Israel does not have a Constitution
  2. Israeli Constitution is like ordinary law
  3. There is no Constitution but a Basic Law of 1950
  4. Israel has a Constitution of 1967.

Answer: c.

66. Israel is a theocratic Jewish state but it follows legal pluralism. Accordingly it has:

  1. Jewish Courts to decide cases of all religious communities.
  2. Secular Courts will apply Uniform Civil Code in respect of all communities
  3. Each religious community has its own Religious Court to deal with Personal law disputes
  4. No distinction between Personal law and Public law.

Answer: c.

Section III – Legal Reasoning

The COVID-19 pandemic has taken the entire world hostage in less than four months, and the global economy has been hit the hardest with governments across the globe implementing stringent policies including lockdown to control the coronavirus outbreak. The pandemic today presents unprecedented challenges and impediments to businesses in conducting their normal operations. The lockdown across the world has caused delays in the performance of contracts and transactions. Now, the question that arises is whether the current situation can enable parties to a contract to alter their obligations with non-compliance of terms neither being regarded as a “default committed by any party” nor a “breach of contract”? There are certain well-accepted practices for dealing with such extraordinary situations in commercial transactions by the inclusion of force majeure & material adverse effect (MAE) clauses. Determination of the types of circumstances so covered by the force majeure clause contained in a contract is essential. Provisions of force majeure often cover natural disasters like hurricanes, floods, and earthquakes as “Acts of God.” Other covered events may include war, terrorism, civil disorder, fire, disease medical epidemics or by reasons of applicable laws or regulations. Broadly, the Courts have interpreted the term “Force Majeure” as an event that can neither be anticipated nor controlled by either of the contracting parties. A force majeure clause applies in the context of ongoing contractual arrangements, whereas, an MAE or material adverse change (MAC) clause applies to the allocation of risk in transactions before their closure or completion. Pandemic and related consequences such as government action is a type of event covered by a force majeure clause, however, its impact on the affected party’s ability to perform its contractual obligations may vary depending upon contractual terms. It is common for force majeure clauses to specify the impact that the event or circumstances in question must have, in order for the clause to be triggered. References may be made, for example, to the event or circumstances having “prevented”, “hindered” or “delayed” performance. These terms require different levels of impact on performance before a party can claim recourse to these clauses. In other words, the force majeure and MAC clauses act as an exception to what would otherwise be treated as a breach of contract. Certain contracts may state that, if a force majeure clause is applied, the contract may automatically be terminated. On the other hand, some contracts may even state that the duty to fulfil the contractual obligation may be suspended for a certain period of time and if the force majeure event is not curbed or treated even after such time, then eventually the contract may be terminated. Though there cannot be a one-size-fits-all solution to this question, and it depends upon how the force majeure clause is worded in a specific contract; and in the absence of the same, applicable laws related to the same will be required to be taken into consideration.

[Excerpt from Business Today, by Ranjana Roy Gawai, April 17, 2020]

67. Based on the Author‘s argument in passage above, which of the following is correct?

  1. Force majeure clauses, generally have a uniform impact on the performance of contracts in all the cases
  2. The impact on the performance of contracts by the usage of force majeure clauses is dependent upon the way such clauses have been constructed in a particular contract
  3. Both force majeure and material adverse change clauses have similar impact on the performance of contracts
  4. All of the above.

Answer: b.

68. Imagine, there is a domestic commercial contract for supply of certain goods for certain price between A and B. However, in pursuance of the same, both A and B forget to negotiate and agree on the terms of a force majeure event and the contractual document does not contain the force majeure clause. In such a situation, what would be the fate of the contract in the event like that of COVID 19?

  1. As the parties did not negotiate on the force majeure clause, either of them cannot take an exception to the breach of contract
  2. The parties can invoke the material adverse change clause
  3. In absence of such clauses in the contract, the Courts may resort to the applicable law, e., Indian Contract Act, 1872 to give relief to the parties
  4. None of the above.

Answer: c.

69. In the same fact situation as mentioned above with a modification that there is a force majeure clause in the contract between A and B, let us suppose, that B, who was to supply goods to A on certain date and time, faced issues in relation to procurement of goods due to mill strike and also because of rise in prices of goods. In this case, can B claim the suspension of performance of contract on the basis of the force majeure clause?

  1. Yes, B can, depending upon the way, the force majeure clause is worded
  2. Such situation cannot be covered under force majeure as it is just a case of disappointed expectations and hence B cannot invoke the clause. It is merely a case of commercial hardship
  3. B can invoke the force majeure clause as the clause is too broad to cover such situations
  4. None of the above.

Answer: b.

70. Typically, the MAE (Material Adverse Change) provision in an agreement contemplates events which if they occur, or are likely occur, would have a “materially adverse change or effect on the assets, business, property, liabilities, financial condition, results, operations of the target” or that “affects the ability of the transacting parties to consummate the transaction” or the “validity orenforceability of the transacting parties to its rights and remedies under the transaction documents”. Which of the following sample clauses in a contract resembles an MAE clause?

  1. “In the event either party is unable to perform its obligations under the terms of this agreement because of Act of God, strikes, equipment of transmission failure or damage reasonably beyond its control, such party shall not be liable for damages to the other for any damages resulting from such failure to perform…”
  2. “Except with respect to payment obligations under this agreement, no party shall be liable for, nor such party shall be considered in breach of this agreement due to, any failure to perform its obligations under this agreement as a result of cause beyond its control, including any earthquake, labour problem, unavailability of supplies…”
  3. Both of the above
  4. None of the above.

Answer: d.

The issue of obscenity has vexed the Courts in India and abroad for a long time now. The intriguing question has always been the same, i.e., what should be the standards to qualify something as obscene in the eyes of law? In the United Kingdom, way back in 1868, the Court laid down the Hicklin test in Regina v. Hicklin (1868 L.R. 2 Q.B. 360), and held that: “The test of obscenity is whether the tendency of the matter charged as obscenity is to deprave and corrupt those whose minds are open to such immoral influences and into whose hands a publication of this sort may fall.” Hicklin test postulated that a publication has to be judged for obscenity based on isolated passages of a work considered out of context and judged by their apparent influence on most susceptible readers, such as children or weakminded adults. However, this test was later rejected by most of the jurisdictions. There were many judgments where it was stipulated by the Indian Courts that obscenity has to be judged in the context of contemporary social mores, current socio-moral attitude of the community and the prevalent norms of acceptability/ susceptibility of the community, in relation to matters in issue. [For example, in Ranjit D. Udeshi v. State of Maharashtra AIR 1965 SC 881; Chandrakant Kalyandas Kakodar v. State of Maharashtra (1969) 2 SCC 687, etc.] These judgments indicated that the concept of obscenity would change with the passage of time and what might have been “obscene” at one point of time would not be considered as obscene at a later period. This came to be known as “Community Standards Test”. In Bobby Art International & Ors. v. Om Pal Singh Hoon (1996) 4 SCC 1, the Court, upholding the Community Standards Test held that, complete message and context of the objectionable scene/firm/picture etc., needs to be examined in order to find out whether the alleged material is obscene or not.

71. A daily local newspaper called ’Ramanand Bazar Patrika‘ having wide circulation in Anandnagar published on 1st July, 2019, an article with a picture of Boris Becker, a world renowned Tennis player, posing nude with his dark-skinned fiancee by name Barbara Feltus, a film actress, which was photographed by none other than her father. The article states that, in an interview, both Boris Becker and Barbara Feltus spoke freely about their engagement, their lives and future plans and the message they wanted to convey to the people at large, for posing to such a photograph. Article picturises Boris Becker as a strident protester of the pernicious practice of “Apartheid”. Further, it was stated that the purpose of the photograph was also to signify that love champions over hatred.

Will the alleged picture classify as an Obscene Material in India?

  1. No, according to the Hicklin test, it will not classify as obscene
  2. Yes, according to the Community Standards Test, the picture will classify as obscene
  3. No, according to the Community Standards Test, the picture will not classify as obscene
  4. Both (a) and (c).

Answer: c.

72. The difference between Hicklin Test and Community Standards Test is:

  1. The former focuses on the susceptibility of the minds of individuals to get corrupted while the later hinges upon the context, intended meaning and contemporaneous socio-cultural environment of the society
  2. As per Hicklin Test, a nude picture of a woman per se can be obscene while as per the later, the picture should be suggestive of deprave mind and designed to excite sexual passion in persons who are likely to see it
  3. The former considers obscenity as a changing concept with changing times while the later does not
  4. All of the above.

Answer: a.

73. The issue of ‘Obscenity’ is fundamentally related with which of the following?

  1. Freedom of religion of an individual
  2. Freedom of speech and expression of an individual
  3. Right to privacy of an individual
  4. All of the above.

Answer: b.

74. Consider the following situations. Choose the correct option as per the Hicklin’s Test.

  1. A movie scene where there are rows of Jewish naked men and women, shown frontally, being led into the Gas Chambers of Nazi Concentration Camp. Not only they are about to die but they are stripped off their basic dignity in the last moments of their life.
  2. The controversial movie scene of Phoolan Devi, the Bandit queen where she is paraded naked and made to draw water from the well within the circle of a hundred men.
  1. 1 is obscene but 2 is not
  2. 2 is obscene but 1 is not
  3. Both 1 and 2 are obscene
  4. Neither 1 and 2 are obscene.

Answer: c.

75. An activist, while being semi-nude, allowed her body to be used as a canvas to paint on by her two minor children who were properly clothed. She uploaded this video of hers on an online platform with a message that she intended to normalise the female form for her children and not allow distorted ideas about sexuality to pervade their mind. An advocate who sees the video, registers a case of obscenity against her.

Is it a case of obscenity as per the Community Standards Test?

  1. This is a pure case of obscenity and she is spreading it
  2. This is a pure case of obscenity as well as child pornography as her children were exposed to her nudity
  3. This is not a case of obscenity because as per the Community Standards Test the video must not be seen in isolation but in the contextual set up of the message that the activist has put on normalisation of a female‘s sexuality
  4. This is a case of obscenity as per the Community Standards Test as the video was blatantly obscene.

Answer: c.

On 7th May 2020, a major leakage of styrene gas was reported from the plastics manufacturing plant ‘LG Polymers’ located on the outskirts of the Visakhapatnam city. The accident took place when the cooling system of a polymers plant got clogged due to the mismanagement of factory workers and resulted in turning the city into a gas chamber. The gas which leaked was styrene gas, which is a ‘hazardous chemical‘ under Rule 2(e) plus Entry 583 of Schedule I of the Manufacture, Storage and Import of Hazardous Chemical Rules 1989.

Principle 1: Polluter Pays Principle

The ‘Doctrine of Polluter Pays’ is a well-established principle of environmental law, which places an obligation of compensating the damage to the people who ought to reimburse it and also have the capacity to disburse it. The principle explicitly affirms that the person who damages or destructs the environment has the absolute obligation to bear the cost of ameliorating the environment. In EnviroLegal Action v. Union of India case, the Apex Court of India held that the polluter is legally responsible to reimburse the individual sufferers as well as pay for the revitalization of the damaged environment.

Principle 2: Principle of Strict Liability

The principle of strict liability was established in the year 1868 in the case of Rylands v. Fletcher, where the Court held that any person who uses his/her land in an ‘unnatural manner’ and who keeps any ‘hazardous substance’ on such premises would be held liable under the principle of strict liability for any ‘damage’ occurred on the ‘escape’ of such perilous substance. However, the person is liable only when there is non-natural use of land; the principle also restricts liability when the escape is due to an act of strangers, Act of God, for example, a natural calamity; due to the person injured or when it happens with the consent of the person injured or with statutory authority.

Principle 3: Principle of Absolute Liability

The absolute liability is a stringent form of strict liability as it is devoid of any exceptions that were mentioned under the earlier principle, was used for the first time in the case of M.C. Mehta v. Union of India. This principle implies that whenever an enterprise is engaged in any dangerous or hazardous activity that threatens the people working in the enterprise and those living nearby, it owes an absolute and non-delegable duty to the community that no harm will be caused. If harm is indeed caused, the enterprise will have to compensate for damages, and can‘t use exceptions provided in the case of strict liability. The enterprise can‘t claim that the harm has not been caused due to negligence (absence of due care) or that it had taken all reasonable precautions.

76. Under which of the following principles, will the company LG Polymers be liable?

  1. Polluter Pays Principle
  2. Strict Liability
  3. Absolute Liability
  4. All of the above.

Answer: d.

77. As per the Polluter Pays Principle, LG Polymers will be liable to pay:

  1. Amount of money for restoration of environment only
  2. Amount of money for restoration of environment and compensation to individuals who suffered the loss
  3. Compensation to the victims
  4. None of the above.

Answer: b.

78. A company ABC Limited operates an industrial chemical plant in the city of Azadnagar. Due to an earthquake on July 22, 2020, the valves of the reactors in the plant get damaged due to which the operators could not properly transport the hazardous gas for chemical vaporisation, resulting in the gas leakage. The gas leakage resulted in the death of 12 workers of the plant and also some people living nearby the plant.

In this situation, in order to decipher the liability of ABC Limited, choose the best option:

  1. ABC Limited can be held liable under the principles of Strict and Absolute Liability and Polluter Pays Principle as well
  2. ABC Limited can be held liable under the Polluter Pays Principle and the Principle of Absolute Liability, but can be exempted under the rule of Strict Liability
  3. ABC Limited will not be held liable under any of the Principles
  4. ABC Limited will be held liable under Strict and Absolute Liability principles but the Principle of Polluter Pays will not be applicable to this situation.

Answer: b.

79. The argument of LG Polymers that, they did not know that the styrene gas could leak:

  1. is a strong argument and can reduce their liability under the Polluter Pays Principle
  2. is a strong argument and can reduce their liability under the Strict Liability Principle
  3. is a weak argument and will not help in reducing the liability of LG Polymers under any of the principles
  4. is a weak argument but may help in reducing their liability under the Strict Liability Principle.

Answer: c.

80. XYZ is a company operating a pesticide factory in the city of Rampur. On one day, due to the negligence of factory staff, there is a leakage of the pesticide gas as a result of which, many pests and insects which feed on the plantation crops in the nearby farm are killed. There is no harm caused to the people living nearby or the workers of the pesticide factory. However, the leakage was so humungous that it reduced the quality of air in the city causing breathing problems for the people living around in the area.

In this case:

  1. XYZ will be absolutely liable but not strictly liable
  2. XYZ will not be liable under the Strict and Absolute Liability Principle because the leakage only killed the pests and insects
  3. XYZ’s liability under the Strict and Absolute Liability Principle will depend upon the inquiry as to whether the leaked pesticide gas was a hazardous substance/activity or chemical or not and; XYZ will be liable under the Polluter Pays Principle
  4. XYZ will not be liable under the Polluter Pays Principle.

Answer: c.

Principle of Natural Justice is derived from the word “Jus Natural” of the Roman law and it is closely related to Common law and moral principles but is not codified. It is a law of nature which is not derived from any statute or constitution. The principle of natural justice is adhered to by all the citizens of civilised State with supreme importance. Natural justice simply means to make a sensible and reasonable decision making procedure on a particular issue. Sometimes, it doesn‘t matter what is the reasonable decision but in the end, what matters is the procedure and who all are engaged in taking the reasonable decision. It is not restricted within the concept of ‘fairness‘, it has different colours and shades which vary from the context. Basically, natural justice consists of three rules.

The first one is “Hearing rule” which states that the person or party who is affected by the decision made by the panel of expert members should be given a fair opportunity to express his point of view to defend himself. Secondly, “Bias rule” generally expresses that panel of expert should be free from bias while taking the decision. The decision should be given in a free and fair manner which can fulfil the rule of natural justice. And thirdly, “Reasoned decision” which states that order, decision or judgment of the Court given by the Presiding authorities with a valid and reasonable ground. The principles of natural justice have been adopted and followed by the judiciary to protect public rights against the arbitrary decision by the administrative authority. One can easily see that the rule of natural justice include the concept of fairness: they stay alive and support to safeguard the fair dealing.

Source: Excerpt taken from blog.ipleaders.in (Dated 12th June, 2019)

81. “Nemo judex in causa sua” or the principle that ‘No one can be a judge in his own case’, relates to which of the following rule of the natural justice?

  1. Bias rule
  2. Reasoned decision rule
  3. Hearing rule
  4. All of the above.

Answer: a.

82. Mr. X is a public servant, employed in a PSU and Mrs. Y who is the wife of Mr. X is also an employee of the same PSU. Ms. A files a complaint against Mr. X for sexual harassment at workplace before the ICC (Internal Complaints Committee), which is headed by Mrs. Y. After completing her inquiry she held that Mr. X is guilty and recommended disciplinary proceedings against him.

Now, on what grounds Mr. X may challenge her findings?

  1. Y is an employee of the same institution and the inquiry should have been conducted by a person outside the institution
  2. The complaint was filed by a female employee, so the inquiry must have been conducted by a male employee
  3. Y is the wife of Mr. X and she holds a personal bias in the case, so the inquiry should not have been conducted by her
  4. X being the accused should have conducted the inquiry himself.

Answer: c.

83. Mr. A is a Judge and he is also the landlord of Mr. B, who resides in an apartment owned by Mr. A. Due to non-payment of rent for 3 consecutive months, Mr. A served an eviction notice to Mr. B. Now, choose the most appropriate option amongst the following:

  1. Being a judge himself, Mr. A cannot issue such notice of eviction
  2. A can issue such notice because it is his duty as a Judge, apart from being a landlord
  3. Such notice is illegal as Mr. B was not given an opportunity to present his case
  4. A issued such notice in his personal capacity as a landlord and not being a Judge.

Answer: d.

84. Which of the following is not in violation of the principles of Natural Justice?

  1. Withholding of an increment of a public servant without giving him an opportunity to defend
  2. Non-renewal of a contract of employment of an employee after the period of contract is over
  3. Initiating a departmental inquiry against a public servant, without giving him an opportunity to submit representation during such inquiry
  4. All of the above.

Answer: b.

85. On the basis of your understanding about natural justice in the passage above, state which of the following is true:

  1. Adherence to the reasoned decision rule is not required if other two rules are complied with
  2. Compliance of bias rule itself rejects the need of the hearing and the reasoned decision rule
  3. Principle of natural justice is incomplete without the compliance of any of its rules
  4. The reasoned decision rule is a substitute for the bias rule.

Answer: c.

Tension prevailed in the Jawahar area in Palghar district after three Mumbai residents, travelling in a Ford Ecosport to Silvassa, were allegedly lynched late on Thursday night. The Kasa police said the incident occurred near Gadakchinchale village under their jurisdiction.”Information received by us indicates that the three occupants of the SUV hailed from Kandivali in Mumbai and were going to attend a funeral in Silvassa,” Superintendent of Police Gaurav Singh, Palghar police said. A large mob of villagers surrounded the car within a matter of minutes and started attacking it with sticks, irons rods and their bare hands, leading to the death of all three occupants. “One of our patrolling vehicles later spotted the severely injured trio lying on the road and stopped to find out the matter. However, our team was also attacked by the mob and the vehicle pelted with stones. Our personnel had to flee and were unable to rescue to the victims,” an officer with the Kasa police said. A wireless alert was sent out later apprising all police stations and units of the incident following which reinforcements were sent to the village and a combing operation was undertaken. “Prima facie information indicates that the trio were mistaken for thieves and attacked. The villagers were on edge due to the ongoing lockdown and unavailability of essential supplies. For the past few days, several rumours have been doing the rounds on social media about thieves and dacoits targeting villages on the highway. As a result, villagers have been patrolling the highway and stopping late night travellers on suspicion,” the Officer said.

Source: Excerpt from The Hindu, written by Alok Deshpande (Dated22nd April 2020)

86. Which of the following statements given below justifies criminalisation of mob lynching?

  1. In a democracy the actual power rests with the people and their actions cannot be penalised by the state
  2. There is no law in India which specifically prohibits mob lynching and therefore it is a justified act of the people
  3. It is impossible to punish the wrongdoer in case of mob lynching
  4. Violence cannot be a tool to implement the will of the majority and such actions are failure of the State.

Answer: d.

87. Which of the following statement justifies the criminalisation of mob lynching as a distinct offence apart from ‘murder’?

  1. The offence of murder is committed by a single individual or a group but mob lynching involves a large group of individuals
  2. When violence is committed by a mob it creates a situation of terror and anarchy having potential to disturb the public peace for a longer period
  3. Mob lynching is an offence which supports the idea of ‘might is right‘, and cannot be accepted in a democratic society
  4. All of the above.

Answer: d.

88. The above passage states about the existence of rumours in the area regarding the thieves and dacoits robbing the villagers or committing child lifting. In the light of this proposition which if the following statement is correct?

  1. The villagers should not be punished as there was a genuine mistake on their part
  2. The villagers should be punished as they do not have any right to punish any person unless he is an offender
  3. The villagers should be punished as no individual in the country is entitled to take law in his own hand and punish the wrongdoer
  4. The villagers should not be punished as they have the right of private defence against such incidents under which they can even kill a person.

Answer: c.

89. In the light of the above passage which of the following incidents of mob violence would be justified?

  1. A group of people committing violence against people who are selling meat of an animal considered to be sacred under their religious beliefs
  2. The residents of a boy‘s hostel caught a drug paddler selling drugs in their hostel and killed him by beating
  3. The villagers injured a woman by pelting stone on her considering her to be witch
  4. None of the above.

Answer: d.

90. Assertion (A): Mob lynching can be justified only in circumstances where the religious sentiments or feelings of a large group of people is associated.

Reason (R): Religious sentiments and feelings are of paramount consideration for the State and should not be interfered with.

  1. Both (A) and (R) are correct
  2. (A) is correct but (R) is incorrect
  3. Both (A) and (R) are incorrect
  4. (A) is incorrect but (R) is correct.

Answer: c.

The Bench of Dr. DY Chandrachud and MR Shah, JJ has refused to transfer to CBI the criminal cases lodged against Republic TV Editor-in-Chief Arnab Goswami for alleged defamatory news show telecast on April 21 in connection with the Palghar mob-lynching case. It also quashed all FIRs against Arnab Goswami except one which was filed in Nagpur and which has been transferred to Mumbai via order dated 24.04.2020.

[Excerpt from SCC Online Blog, May 19, 2020]

Delivering the verdict, Justice Chandrachud said “ Article 32 of the Constitution constitutes recognition of the constitutional duty entrusted to this Court to protect the fundamental rights of citizens. The exercise of journalistic freedom lies at the core of speech and expression protected by Article 19(1)(a). The petitioner is a media journalist. The airing of views on television shows which he hosts is in the exercise of his fundamental right to speech and expression under Article 19(1)(a). India’s freedoms will rest safe as long as journalists can speak truth to power without being chilled by a threat of reprisal…Free citizens cannot exist when the news media is chained to adhere to one position. Yuval Noah Harari has put it succinctly in his recent book titled ‘21 Lessons for the 21st Century’: “Questions you cannot answer are usually far better for you than answers you cannot question.”

[Excerpt from Arnab Ranjan Goswami vs Union Of India on 19th May, 2020]

91. The above passage mentions about the journalist‘s request to transfer the case to the CBI (Central Bureau of Investigation) for investigation. What could be the appropriate reason for such request?

  1. The CBI is a central investigating agency and conducts more scientific investigation
  2. Where State government is an interested party, the investigation conducted by the State police, may be biased
  3. The FIRs relate to an offence of defamation of the President of a national party, so the matter should be investigated by the CBI
  4. None of the above.

Answer: b.

92. The above passage quotes the observation of the Court in relation to freedom of speech and expression as, “Questions you cannot answer are better than questions you cannot question.” Now, based on such observation, chose the most appropriate option which describes the scope of such questions.

  1. The term “questions” mentioned are not subject to limitations imposed upon the freedom of speech and expression
  2. Those questions are limited to the information warranted by the person
  3. The Court is referring to the right to information through the above quote
  4. The questions are also subject to limitations imposed upon the freedom of speech and expression.

Answer: d.

93. It is an established principle of law that the offence of defamation (harming the reputation) has many exceptions and the imputation of truth is one of those, but the accused must prove that such imputation was true and also for the public good. In the light of this statement which of the following questions/statements are not defamatory?

  1. An article published in a newspaper about the unaccounted property of a public servant in the form of a questionnaire
  2. A question to a female actress about her sexual relationship with a married man
  3. A question to a public servant about his extramarital relationship with a colleague
  4. An Article published in a magazine stating a female politician as ‘B grade‘ actress.

Answer: a.

94. In the above passage, the Court ordered for the merging of different FIRs into one and the investigation to be conducted at Mumbai. What is the reason behind such order?

  1. Multiple FIRs mean multiple investigations and the same might cause hardships to the accused
  2. The first FIR was registered at Mumbai and the investigation should be conducted at Mumbai
  3. The accused resides at Mumbai and the investigation should be conducted only at Mumbai
  4. Multiple FIRs are frivolous and there is only one cause of action.

Answer: a.

95. Defamation is punishable:

  1. Both as a civil wrong as well as a crime
  2. Only as a civil wrong
  3. Only as a criminal wrong
  4. None of the above.

Answer: a.

Common intention implies a pre-arranged plan and acting in concert pursuant to the plan. Common intention comes into being prior to the commission of the act, which need not be a long gap. To bring common intention into effect a pre-concert is not necessarily be proved, but it may well develop on the spot as between a number of persons and could be inferred from facts and circumstances of each case. For example A and B caught hold of C where only B stabbed C with a knife but A is also liable for murder as there was a pre concerted action. In the case Pandurang v. State of Hyderabad, Supreme Court emphasized on this point that prior concert need not be something always very much prior to the incident, but could well be something that may develop on the spot, on the spur of the moment.

Common Intention and Similar Intention

Common intention does not mean similar intention of several persons. To constitute common intention it is necessary that the intention of each one of them be known to the rest of them and shared by them. In the case of Dukhmochan Pandey v. State of Bihar, the Supreme Court, held that: “Common intention which developed at the spur of the moment is different from the similar intention actuated a number of person at the same time….the distinction between a common intention and similar intention may be fine, but is nonetheless a real one and if overlooked, may lead to miscarriage of justice….” The mere presence of accused together is not sufficient to hold that they shared the common intention to commit the offence in question. It is necessary that the intention of each one of ‘several persons‘ be known to each other for constituting common intention.

96. A gang of six members went to a bank, armed with weapons to commit a heist. While five of the gang members went inside the bank, Mr. A (the sixth member) waited outside the bank to alert them on any threat. During the heist one of the gang members fired a gun at the branch manager, as a result he died. All five escaped but Mr. A was caught and arrested.

Now, choose the most appropriate option as per the principle stated in the above passage.

  1. A is not liable for murder as he was outside the bank and there was no common intention
  2. A along with all other members of the gang are liable for murder as there was common intention
  3. Only that person is liable for murder who actually fired the gun
  4. A is liable only for the heist and no other offence.

Answer: b.

97. Raman and Raghav were riding on a motorcycle on a busy street, suddenly Aman (another biker) bumped into their bike. A heated argument started between the three of them. While Raghav started abusing Aman, Raman hit Aman with an iron rod lying on the road and as a consequence he died. Now, chose the correct option.

  1. Both Raman and Raghav are liable for murder as there was a common intention developed on the spot
  2. Raghav is not liable for murder as there was no common intention to kill Aman
  3. No one is liable as Aman was a wrongdoer himself and he started the fight
  4. Only Raghav is liable for murder as he started abusing Aman.

Answer: b.

98. After reading the passage which of the following is not correct in relation to the difference between common and similar intention?

  1. Similar intention is developed prior to the commission of offence but the common intention is developed only at the time of commission of offence
  2. Under common intention each of the offender is equally liable for the offence but under similar intention each of the offender is differently liable
  3. In order to determine the existence of similar or common intention, one must analyse the fact and circumstances of each case
  4. The boundary between similar and common intention is very fine and it may sometime overlap.

Answer: a.

99. Mr. X and Mr. Y entered into a house at night to commit theft, while committing theft Mr. Y committed sexual assault on a minor girl of aged 11 years. Identify for which of the following offences Mr. X is liable for.

  1. Both theft and sexual assault as there was a common intention
  2. Only theft as there was a similar intention
  3. Only theft as Mr. X had a different intention from Y
  4. He would not be liable for any offence.

Answer: c.

100. Which of the following statements is correct in relation to the difference between common intention and similar intention?

  1. The intention of the accused and co-accused can be inferred from the facts and circumstances of each case
  2. Under common intention, it is considered that all the accused have jointly committed the offence themselves and are jointly liable
  3. Each accused is liable for the offence he has actually committed, if the common intention cannot be proved
  4. All of the above.

Answer: d.

Article 20(1) of the Indian Constitution prohibits Ex post facto laws. The expression ”Ex post facto law” means a law, which imposes penalties or convictions on the acts already done and increases the penalty for such acts. In other words, Ex post facto law, imposes penalties retrospectively. For example, the Dowry Prohibition Act, 1961 came into force from 20.5.1961. A person guilty of accepting dowry is punishable under the Act after 20.5.1961 and not before 20.5.1961.

Ex post facto laws are of three kinds as follows:

(a) A law which declared some act or omission as an offence for the first time after the completion of that act or omission.

(b) A law which enhances the punishment or penalty for an offence subsequent to the commission of that offence.

(c) A law which prescribes a new and different procedure for the prosecution of an offence subsequent to the commission of that offence.

Clause (1) of Article 20 provides protection only in respect of the above first two categories of ex post facto laws, i.e., laws which declare acts as offences subsequent to the commission to those acts and laws which enhance the penalty subsequently.

Article 20(1) provides: No person shall be convicted of any offence except for violation of a law in force at the time of the commission of the act charged as an offence, nor be subjected to a penalty greater than that which might have been inflicted under the law in force at the time of the commission of the offence. The first part of clause (1) provides that no person shall be convicted of any offence except for violation of law in force at the time of the commission of the act charged as an offence. The second part of clause (1) protects a person form a penalty greater than that which he might have been subjected to at the time of the commission of the offence.

101. The Parliament of India legislated the Sexual Harassment at Workplace Act, in the year 2013 and simultaneously some new offences were also added to the Indian Penal Code, 1860. Mr. A is an employee of an institution, against whom departmental enquiry has been initiated for committing sexual harassment of a female colleague in the year 2012. Now, choose the correct option.

  1. Such inquiry is valid as per Article 20 (1) of the Constitution as the complaint was made after the Act came into force
  2. Article 20 (1) will not be applicable as the matter does not relate to an offence
  3. As the Act was passed in the year 2013, any inquiry under such Act is invalid
  4. Only inquiry may be conducted but no penal action can be taken against him after such inquiry.

Answer: d.

102. Article 20 (1) would not affect which of the following acts of the legislature?

  1. Act of the legislature enhancing the term of imprisonment
  2. Act of the legislature enhancing the amount of fine
  3. Act of the legislature changing the punishment of death to life imprisonment
  4. Act of legislature changing the nature of imprisonment from simple to rigorous.

Answer: c.

103. Mr. A is a student of law, aged 19 years. He is socially active and expresses his opinion on every social and political event of the nation through social media platforms. In one of his blog, he severely criticised the policy of a State Government of changing names of cities and towns. He also stated that the government is biased towards a particular religion. The said blog was posted on 19th April, 2020 and subsequently, an amendment was made to Indian Penal Code whereby ‘Hate Speech‘ was made a distinct offence and punishment was prescribed. An action was brought against him under the said provision for the blog. Now, choose the most appropriate option amongst the following:

  1. A may be liable for the offence of hate speech as the blog was not removed even after the amendment
  2. A may be liable for the offence of hate speech as Article 20 (1) does not cover such areas
  3. A may not be liable for the offence because the act was done before the amendment
  4. A may not be liable for the offence as his blog was innocent and a fair criticism.

Answer: a.

104. Considering the fact situations given in the above question, the Parliament passes a legislation in September 2020, whereby an amendment is made to the Juvenile Act and now a person below the age of 20 would be a juvenile and special procedure would be followed for his trial. Choose the most appropriate option amongst the following:

  1. A would not be considered as juvenile as it is prohibited under the scheme of Article 20 (1).
  2. A would not be considered as juvenile because the amendment came after he committed the offence
  3. A would be considered a juvenile and tried under the new procedure
  4. None of the above.

Answer: c.

105. Considering the fact situation in the third question to this passage, the Parliament passes a legislation for the Probation of Offenders, under which any offender below the age of 21 will not serve the sentence of imprisonment in a prison, instead he will serve the sentence in a probation house. Now, choose the most appropriate option.

  1. A will get the benefit of Probation of Offenders Act
  2. A will not get the benefit of Probation of Offenders Act as it is prohibited by Article 20 (1) of the Constitution
  3. It is discretion of Mr. A to decide whether he wants such benefit or not
  4. None of the above.

Answer: a.

Section IV – Logical Reasoning

Some readers wondered whether my reading of the crisis in the news media is about journalism or about the news industry. Though the fortunes of the news industry have a bearing on journalism, there is, indeed, a difference between looking at issues that govern journalism and the factors that contribute to the financial stress of the media industry. Over the last few years, I have been discussing the impact of digitalisation on journalism. The pandemic has accelerated the process of digital transformation of the news media. Therefore, these issues need close scrutiny. One of the defining elements of analog journalism was the way two crucial functions of journalism — bearing witness and making sense — complemented each other and helped people make informed choices. When I talk about the strength of analog, I am neither romanticising the past nor am I a Luddite. Most importantly, I do not believe in nostalgia. Many scholars have established “how rose-coloured glasses always leads to an unfair distortion — looking back on the best of the past while comparing it to the worst of the present.” Hence, when I talk about the digital information news environment, I am talking about how there needs to be a conducive atmosphere for credible information to resonate with the people. Literature on misinformation, disinformation and mal-information reveals a new distinction in the minds of the citizens. Editors and journalists have to contend with a new breed of sceptics. These are the people who trust social media forwards more than evidence-based, data-driven journalism. Their confirmation bias flows from encrypted social media platforms that are full of conspiracy theories.

[Excerpt from an Article by A.S. Panneerselvan, The Hindu, dated August 10, 2020]

106. Which of the following conveys the main idea expressed in the passage?

  1. Those who subscribe to newspapers are sceptics
  2. Those who subscribe to newspapers and news channels often contribute in distortion of the truth
  3. Social media journalism distorts the truth
  4. All of the above.

Answer: c.

107. In this line, “Hence, when I talk about the digital information news environment, I am talking about how there needs to be a conducive atmosphere for credible information to resonate with the people” the Author assumes that:

  1. The digital news is neither evidence based nor data driven
  2. The digital journalism does not help people in making informed choices
  3. Both (a) and (b)
  4. The impact of digital journalism has created an informed platform of journalism.

Answer: c.

108. Which of the following strengthens the Author‘s argument?

  1. Conspiracy theories have become an alternative truth
  2. The pandemic is the only reason behind the dependence of people on social media journalism
  3. The News industry is witnessing a tremendous change
  4. None of the above.

Answer: a.

109. What does the Author indicate when he quotes that “Rose coloured glasses always lead to unfair distortion”?

  1. Digital Journalism is twisted
  2. Author means to say that he does not have any disproportionate bias for analog journalism
  3. Rosy picture of every news cannot be created
  4. All of the above.

Answer: b.

110. The Author has severely targeted and criticized which one of the following factors of Digital Journalism?

  1. Social Outreach
  2. Credibility of Information
  3. Lack of Information
  4. None of the above.

Answer: b.

In 1985, Mikhail Gorbachev, launched an ill-fated anti-alcohol campaign in the then Soviet Union. The anti-alcohol campaign had some beneficial public health consequences: Crime fell and life expectancy rose. But the campaign was a political and economic disaster. Gorbachev forgot that the addiction of the State to alcohol revenue was even more incurable than the addiction of some citizens to alcohol itself. The budgetary losses created an economic crisis. Historians suspect that more than the loss of the Soviet Empire, it was this campaign that delegitimised Gorbachev. An old Soviet joke went like this: A disaffected and angry citizen, fed up of standing in lines for vodka, decided to go assassinate Gorbachev. He soon came back and ruefully reported that the lines to assassinate Gorbachev were even longer than the lines for Vodka. As the lockdown eased in India, and social distancing went for a toss at alcohol outlets, we were reminded of how difficult an issue alcohol is to rationally discuss in India. The stampede was caused by the ineptness with which the opening was handled in most cities. Alcohol has also migrated from being a question of personal freedom and choice to an issue in broader cultural wars, an odd site on which we measure progressivism in India. It is also a window on how liberalism has been misunderstood. Liberals should, rightly, be suspicious of prohibition on moral and practical grounds. Government grossly exceeds its legitimate power when it interferes with the rights of individuals to lead their lives as they please, and fashion their selves after their own ideals, interests and preferences. And certainly, moralism or puritanism on alcohol cannot be the basis of state policy. That moralism has no basis, and it violates the dignity and freedom of individuals.

[Excerpt from an Opinion by Bhanu Pratap Mehta, The Indian Express, May 7, 2020]

111. In the above passage, the author uses a/an …….. to strengthen his argument.

  1. Metaphor
  2. Analogy
  3. Personification
  4. None of the above.

Answer: b.

112. Which of the following can be inferred from the passage above?

  1. State must not interfere at all with the individual‘s right to drink
  2. Liberalism has always been misunderstood
  3. It is very difficult to discuss any issue relating to personal freedom of individuals with rationality in India
  4. None of the above.

Answer: d.

113. Which of the following statements weakens the argument that Moralism on Alcohol cannot be the basis of State Policy?

  1. The State should not interfere with people‘s right to drink; but there will be a backlash if drinking takes forms that inflict great social harms
  2. Freedom should not be divorced from Moderation
  3. The State should not interfere in matters of sexuality or intimacy. But norms of freedom will impose serious costs and will not survive if the expressions of sexuality are consistently degrading or violent, as we have seen in the locker room scandals.
  4. All of the above.

Answer: d.

114. Which of the following is true as per the passage above?

  1. Liberals should not be concerned about prohibition on alcohol at all
  2. Moralism should be based on justified ideals
  3. Moralism must not be the basis of any State action
  4. All of the above.

Answer: b.

115. According to the passage, which of the following could be part of State‘s policy to regulate alcohol use in India?

  1. Ban on alcohol shops.
  2. Education on intelligent drinking.
  3. Regulating outlet density of alcohol shops.
  4. Community Intervention Policy.
  1. I, II and III
  2. II and IV
  3. II, III and IV
  4. I, II, III and IV.

Answer: c.

Don‘t miss the cloud behind the silver lining. The Class XII CBSE pass result has soared to an all-time high of 88.8%. The number of students scoring 95% and above has more than doubled. If only these higher marks were a reflection of students getting more skilled and more competitive. Instead, the odds are they simply mirror an assessment scheme relaxed to compensate students for the many disruptions originating in the pandemic. The goal of reducing student stress levels during this turbulent time is excellent and laudable. But it is a fallacy that high marks can accomplish this by themselves. The real stressor, after all, is shortage of opportunities. When every bout of grade inflation raises cut-offs for higher education even higher, it is no succour. That India‘s Gross Enrolment Ratio is only 27% compared to Indonesia‘s 36%, Thailand‘s 49% and the US‘s 88% is just one measure of the toll taken by our failure to build adequate colleges and universities. Licence raj continues to suppress autonomy and expansion in this sector. For example, a new national education policy is reportedly mulling at least 20% of students being able to attend private higher educational institutions through freeships and another 30% through scholarships, besides fee caps. On top of the stasis already wreaked by reservations, this would be disastrous. Browbeating the private sector to make up for public sector deficiencies is counterproductive. As the pandemic has underlined the key role of good government services in healthcare, so too is raising the standard of publicly funded schools and universities essential. What the students need is an ecosystem where government institutions deliver quality education and private options are plentiful. It is better prospects alone that will best alleviate students‘ stress. Meanwhile, crudely chopping syllabi will only worsen their disorientation and should be reconsidered.

[TOI Edit, Times of India Editorials, Dated July 15, 2020].

116. What is the ‘cloud behind the silver lining‘ in the passage above?

  1. The pandemic which is impacting the current education system
  2. That, the Governmental educational institutions are not able to deliver quality education
  3. Scoring higher marks or percentage in school education does not guarantee skilled and competitive candidature of the student
  4. License raj has negatively impacted higher education.

Answer: c.

117. Which of the following statements weakens the argument that chopping the syllabi in the times of Pandemic will only worsen a student‘s disorientation and hence should be reconsidered?

  1. It does not matter even if syllabi is chopped. The students who don‘t want to study will even not study in the minimal syllabi structure even during the pandemic
  2. Students don‘t care about syllabi. They study only for marks and grades
  3. Reduced syllabi will help in achieving the teaching-learning targets in due time and hence won‘t put pressure on students
  4. All of the above.

Answer: d.

118. The number of students scoring 95% and above in XII class board exams has more than doubled. Which of the following statements serves as a plausible explanation for the same?

  1. Students have started studying with sincerity and concentration
  2. Evaluation of answer scripts has become liberal
  3. Question papers these days are easier to solve
  4. All of the above.

Answer: d.

119. For developing a student friendly ecosystem of education, which of the following serve as need of the hour, as per the Author?

  1. Increasing the syllabi in school education.
  2. Building adequate number of public colleges and universities.
  3. Eliminating reservation in admissions to higher education system.
  4. Creating a quality based education system.
  1. Only I and II
  2. Only I, II and IV
  3. Only III
  4. Only II and IV.

Answer: d.

120. Which of the following can be inferred from the passage above?

  1. Scoring high marks in XII class can help in reducing stress amongst students in the times of pandemic
  2. High cut-off of marks also contribute in reducing opportunities for the students
  3. Private institutions should not be regulated
  4. License raj should be done away with.

Answer: b.

If anything, the novel corona virus disease (COVID-19) pandemic has taught us to rethink our lifestyles and question our need to travel to work every day. After all, in the age of internet, zoom meetings and webinars can be virtually conducted and physical distancing is possible. A large nature of work in cities is of tertiary nature, a major part of which can be done from home. This can affect the way offices function and reduce the need for all employees to be physically present every day. Information Technology companies are already contemplating a move of making many of its employees work from home and make this a ’new normal‘. If more firms follow suit, the need for huge office buildings and central business districts would change. Apartments cannot get bigger at the same rate, and people may not have the space or atmosphere to work from home. This would mean that more people would opt for co-working spaces close to their homes. This could be a game changer, for it would provide the cities to better distribute their activities throughout the spaces and rid themselves of the idea of zoning. Only a handful of professionals are allowed to function from residential zones, including doctors, lawyers, architects, etc. This needs serious rethinking. Many more professions of similar nature that do not disturb surrounding residence and have no requirements of special services should be added to the list. This new work culture would bring associated demand for food joints, cafés, stationery shops, etc.

[Excerpt from “Down to Earth” Magazine, Blog authored by Sugeet Grover, August 17, 2020]

121. Which of the following is the main idea behind the passage?

  1. Work from home is the new normal
  2. Work from home will result in shutting down of big buildings
  3. Work from home culture will result in systematization of City planning and development
  4. More professionals should be included in the ‘work from home‘ list.

Answer: c.

122. Which of the following can be inferred from the passage above?

  1. The ‘work from home‘ culture is a no-chaos culture
  2. There is lack of distribution of activities in the ‘work from home‘ scenario
  3. The routine culture as opposed to ‘work from home‘ does not benefit the society in any way
  4. All of the above.

Answer: a.

123. Which of the following can be associated benefits of the new culture?

  1. It will save commute time
  2. The city‘s land can be used more efficiently
  3. Better distribution of activities would be associated with safer streets and public spaces as they would not go unused and hence have less likelihood of being used for unlawful activities
  4. All of the above.

Answer: d.

124. Consider the following statement: “Work from home culture is a sustainable method of business and should be extended to all professions.” Which of the following weakens the Author‘s argument?

  1. Not every type of profession fits in the ‘work from home‘ culture as different professions have different requirements and need different environmental and work space settings
  2. ‘Work from home‘ impacts a person‘s mental health
  3. Both (a) and (b)
  4. Neither (a) nor (b).

Answer: c.

125. Which of the following weakens the author’s argument that ‘work from home’ culture will enable better city planning?

  1. Development of co-working spaces will further create chaos in terms of land planning
  2. City planning is not only about reducing spaces
  3. Work from home will also reduce the flow of money in the economy
  4. All of the above.

Answer: d.

Directions: Following is an array of questions to test your reasoning ability in different situations. Answer each of them according to the question asked in each of them respectively:

126. If highways were restricted to cars and only those trucks with capacity of less than 8 tons, most of the truck traffic would be forced to run outside highways. Such a reduction in the amount of truck traffic would reduce the risk of collisions on highways. The conclusion drawn in the first sentence depends on which of the following assumptions?

  1. The roads outside highway would be as convenient as highway for most drivers of trucks
  2. Most of the roads outside highways are not ready to handle truck traffic
  3. Most trucks that are currently running in highway have a capacity of more than 8 tons
  4. Cars are at greater risk of being involved in collisions than are trucks.

Answer: c.

127. Read the debate between Harry and Potter and identify the main issue:

Harry: Within democracies, voters are entitled to know the down-side to a candidate and the other side is obviously well placed to voice it. To stifle one‘s ability to voice negative things about a candidate would be to obstruct democracy and limit free speech.

Potter: Negative advertisements produce the politics of the personal, since an easiest advert is an attack-advert which focuses on the personality or personal qualities of one‘s opponent. A negative advertisement is one that focuses upon a rival product, in this case, a rival election candidate or party in order to point its flaws and to persuade the public to not vote for it.

  1. Whether negative advertisement strengthens democratic governance?
  2. Whether the practice of negative advertisements is good for democracy and politics?
  3. Whether negative advertising needs to be banned?
  4. Whether negative advertising produces the politics of the personal?

Answer: b.

128. Which alternative applies to the following Statement and Assumption?

Statement: Go by Aeroplane to reach Delhi from Chennai quickly.

Assumptions:

  1. Chennai and Delhi are connected by Air service.
  2. There is no other means to commute from Chennai to Delhi.
  3. The Air distance between Delhi and Chennai is less.
  1. Only I is implicit
  2. Only I and II are implicit
  3. Only I and III are implicit
  4. Only II is implicit.

Answer: c.

129. Ananya and Krishna can speak and follow English. Bulbul can write and speak Hindi as Archana does. Archana talks with Ananya also in Bengali. Krishna cannot follow Bengali. Bulbul talks with Ananya in Hindi. Who can speak and follow English, Hindi and Bengali?

  1. Archana
  2. Bulbul
  3. Krishna
  4. Ananya.

Answer: d.

130. If ‘367’ means ‘I am happy’; ‘748’ means ‘you are sad’ and ‘469’ means ‘happy and sad’ in a given code, then which of the following represents ‘and’ in the code?

  1. 3
  2. 6
  3. 9
  4. 4.

Answer: c.

One of the biggest casualties of the Covid-19 pandemic and the resultant lockdown has been institutionalised education. Schools have been shut to prevent the spread of the virus and this has given way to online classrooms, a very new concept in India even for the most sophisticated schools. It is commendable how easily some educational institutions have moved to virtual classrooms, all thanks to tools such as Zoom, Google Hangouts and Microsoft Teams. But there are some still struggling to get online. The online classes, whatever the enabling technology, is only as good as the teachers and the ability of the students to grasp the new teaching technique. One of the teachers felt students are actually more responsive and active in online classrooms, compared to when they are in physical ones. “This could be because this is a new concept and they are excited to explore it with the teachers. They also don’t get distracted by their classmates, which frequently happens in a regular class.” Teachers do find the absence of a blackboard a disadvantage and network connectivity a constant problem.”We miss the clarity that a blackboard gives us, we are kind of making do with the virtual whiteboard on Zoom.” Shweta Kawatra, a parent who teaches in a New Delhi school, highlights that many students have not been able to take advantage of the virtual platform because they do not have a suitable device at home or lack a good internet connection. “It has its own share of disadvantages too. Too much screen time can be perilous for health. Prolonged online sessions can be overwhelming and may lead to problems related to vision, body posture and sleep disorder,” Kawatra adds.

[Excerpt from an Article by Sneha Saha, The Indian Express, dated April 18, 2020]

131. What is the central idea of the passage above?

  1. Institutionalised education was much better than the current online education system
  2. Institutionalized education cannot be replaced by any other system of education
  3. Virtual classroom teaching is a sustainable approach to education in modern times
  4. None of the above.

Answer: d.

132. Which of the following statements cannot be inferred from the passage above?

  1. Virtual classroom teaching is a sustainable approach to education
  2. Virtual classroom teaching causes health problems
  3. Institutionalized education system has its advantages
  4. All of the above.

Answer: a.

133. Which of the following is an implicit assumption in the observation that students are more active and responsive in online classrooms as compared to that in physical ones?

  1. Students are not serious while studying in physical classrooms.
  2. Online classrooms inculcate more interest in students towards studies than the physical classrooms.
  3. Students don‘t like physical classrooms at all.
  4. Students sleep in physical classrooms.
  1. Only I and II
  2. Only III and IV
  3. I and IV
  4. I, II, III and IV.

Answer: a.

134. Which of the following can be inferred from the passage above?

  1. There is a divide in opinions of people on the merits and demerits of Online education
  2. Online education system is going to replace the Institutionalized education system
  3. The institutionalized education system is not at all perilous for health of students
  4. All of the above.

Answer: a.

135. Consider this statement: ‘Virtual education provides for various online seminars, opportunities to interact with learned experts, pro bono education classes etc., for students across India irrespective of their economic status and background.‘

Which of the following statements weakens the statement?

  1. Many students in remote areas do not have access to internet facilities.
  2. Many students in rural areas do not have devices to take advantage of the Virtual classrooms.
  3. Online Seminars and pro bono classes are not interesting and do not cater to the needs of students across India.
  4. The mobile phone data packages to access such heavy online contents are not enough and participation in such events involves costly data packages which is dependent upon the economic status of those students.
  1. Only I, II, III
  2. Only III and IV
  3. Only I, II and IV
  4. Only I and II.

Answer: c.

Section V – Quantitative Techniques

The following data presents the cases of Japanese Encephalitis in five states of India.

136. Number of females affected by Japanese encephalitis in West Bengal are what percentage (up to 1 decimal) more or less than that of females affected in Odisha?

  1. 6 % less
  2. 7 % more
  3. 8 % less
  4. 8 % more.

Answer: a.

137. What is the average number of males affected by Japanese encephalitis in all the five States?

  1. 9,000
  2. 13,500
  3. 18,400
  4. 20,600.

Answer: d.

138. If the total number of males affected by Japanese Encephalitis in other States are 87 % more than the average number of males in Odisha, West Bengal, Chhattisgarh and Jharkhand, the total number of males affected in India is:

  1. 44,880
  2. 1,40,880
  3. 1,47,880
  4. 68,880.

Answer: b.

139. By what percentage is the average of males in Chhattisgarh, Jharkhand and Andhra Pradesh together more or less than the average of females in West Bengal, Jharkhand and Odisha together who are affected by Japanese encephalitis?

  1. 22
  2. 24
  3. 26
  4. 28.

Answer: c.

140. What is the percentage (up to 1 decimal) of females affected by ‘Japanese encephalitis‘ of the total affected population across five States?

  1. 25.4
  2. 36.7
  3. 24.5
  4. 37.6.

Answer: d.

A room having the dimensions as mentioned in the chart below has a table, a bed and a chair. There are three books kept on the table (side by side) and also a lamp having radius, 3.5 inch. Now, on the basis of information given below answer the following?

Sr. No. Object Length Breadth Height
1. Room 18 feet 12 feet 12 feet
2. Bed 3.5 feet 6 feet 3 feet
3. Table 3 feet 4 feet 4 feet
4. Chair 2 feet 2.5 feet 4 feet
5. Books 12 inch 6 inch 3 inch

141. What would be the area of the room which is not covered by any object as mentioned above?

  1. 176.5 sq. ft
  2. 178 sq. ft
  3. 367 sq. ft
  4. 245.5 sq. ft.

Answer: b.

142. What is the area of the table which is uncovered by any of the objects?

  1. 1656 sq. inch
  2. 1617.52 sq. inch
  3. 1473.52 sq. inch
  4. 1512 sq. inch.

Answer: c.

143. How many tables are required to cover the floor surface of the room completely?

  1. 18
  2. 11
  3. 21
  4. Data inadequate.

Answer: a.

144. If the entire room is to be filled by books having same dimensions as lying on the table then how many books are required.

  1. 2,00,736
  2. 20,736
  3. 11,736
  4. 17,522.

Answer: b.

145. How many chairs can be put inside the room so as to cover the entire floor area of the room?

  1. 40
  2. 21
  3. 43
  4. 18.

Answer: c.

Direction: In the figure given below, a goat is tied to a pole (at point O) which is the center of the semi -circular plot. The area of the plot is equal to its perimeter. BO is the length of the rope through which the goat is tied to the pole. Answer to the closest decimal.

146. The length of the rope is

  1. 2 + π
  2. 2 + π/2
  3. 2 + 2/ π
  4. 2.

Answer: c.

147. The area of the plot is

  1. 18.13
  2. 21.83
  3. 28.13
  4. 11.83.

Answer: b.

148. If a triangle is created by joining points A, B, and C. What will be the length of AB?

  1. 3.73
  2. 7.33
  3. 1.86
  4. Can’t be calculated with the given data.

Answer: a.

149. What will be the area of the triangle ABC?

  1. 6.96
  2. 9.66
  3. 3.66
  4. Can’t be calculated with the given data.

Answer: a.

150. What will be the perimeter of the triangle ABC?

  1. 40.4
  2. 30.3
  3. 20.2
  4. 10.1.

Answer: d.

Common Law Admission Test | CLAT UG 2020 | Click Here to Download PDF


  1. Law Library: Notes and Study Material for LLB, LLM, Judiciary and Entrance Exams
  2. Legal Bites Academy – Ultimate Test Prep Destination
Updated On 16 Oct 2021 5:49 AM GMT
Admin Legal Bites

Admin Legal Bites

Legal Bites Study Materials correspond to what is taught in law schools and what is tested in competitive exams. It pledges to offer a competitive advantage, prepare for tests, and save a lot of money.

Next Story